6
$\begingroup$

Suppose we are interested in the sum

$\sum _{n\leq x}a_n.$

The study of the sum

$\sum _{n\leq x}a_n\log (x/n)$

may be easier.

What can one say about the first sum from knowing the behaviour of the second?

In the case I have in mind, I have

$\sum _{n\leq x}a_n\log (x/n)=x+\mathcal O\left (x^{1/2}\right )$

and would like a similar result for the sum without weights.

How feasible is this? I'm quite sure an asymptotic formula holds for the sum without weights, but I don't know if I should expect to lose a power saving. (Logarithms and epsilons in the error are not of concern.)

I'm not sure which properties exactly of the $a_n$ are important. It may be useful that they are all non-negative.

Any pointers as to what one should expect in such a situation would be very much appreciated.

If it seems that one can't really say much without knowing at least something more about the $a_n$ I can give some more details. But I think my problem has more to do with the fact that I'm lacking some general principles in dealing with weights, so I'll leave the $a_n$ arbitrary for now.

Thanks very much in advance.

$\endgroup$
8
  • 5
    $\begingroup$ The standard technique is partial summation. More precisely, if $S(x)=\sum_{n\leq x}a_n\log n$, then the first sum is $\int_{2-}^x\frac{dS(t)}{\log t}$, which can be manipulated by integration by parts (see en.wikipedia.org/wiki/…). Have you tried this? $\endgroup$
    – GH from MO
    Sep 25, 2015 at 17:49
  • 1
    $\begingroup$ Another idea is to compare the Mellin transforms of the two summatory functions: the transforms are very close to each other. I don't have time to elaborate on this now. $\endgroup$
    – GH from MO
    Sep 25, 2015 at 17:58
  • $\begingroup$ I tried the "amateur version" of summation by parts (not thinking in terms of Riemann-Stieltjes integrals), but dismissed it almost immediately since the term $\sum _{n\leq t}a_n\log (x/n)$ comes up, which we don't understand. But I don't know if this is a correct objection, since I don't know "philosophically" why it doesn't work. (In your example, after applying integration by parts, the sum $\sum _{n\leq t}a_n\log n)$ comes up, which we don't understand either.) $\endgroup$
    – tomos
    Sep 25, 2015 at 20:04
  • $\begingroup$ Mellin transforms: I'm sort of going the other way. I tried to study the first sum through its Mellin transform, but wasn't successful. However, I could do the second sum with the Mellin transform approach. (My thinking was: "well, if I can do it with logs then the whole sum shouldn't really be any bigger so it should be fine" - perhaps I oversimplified and didn't understand that the problem without the logs is significantly harder, even though it's not much bigger - is this correct?) $\endgroup$
    – tomos
    Sep 25, 2015 at 20:16
  • 1
    $\begingroup$ Hey so it looks like you're trying to use a truncated Perron formula --- have you checked the standard 'Perron formula with error term' that would e.g. appear in Montgomery-Vaughan? As for the weights, $\sum_{n\leq x+x^{3/4}} a_n\log((x+x^{3/4})/n) - \sum_{n\leq x} a_n\log(x/n) = (\sum_{n\leq x} a_n)\log(1+x^{-1/4}) + \sum_{x < n\leq x+x^{3/4}} a_n\log((x+x^{3/4})/n)$. The latter sum, if $|a_n|\ll 1$, is $\ll x^{1/2}$. The left-hand side, by hypothesis, was $x^{3/4} + O(x^{1/2})$. Thus (rearranging) $\sum_{n\leq x} a_n = x + O(x^{3/4})$. Hope I haven't made a mistake in the above! $\endgroup$
    – alpoge
    Sep 26, 2015 at 6:59

1 Answer 1

7
$\begingroup$

Put $A(x) =\sum_{n\le x} a_n$, and $B(x) =\sum_{n\le x} a_n\log x/n$. Then $$ B(x) = \int_1^x A(t)\frac{dt}{t}. $$ So information about $A(x)$ readily translates to information about $B(x)$ and there is no loss since integration (which makes things smoother) is involved. But to pass from $B(x)$ to $A(x)$ we need to differentiate, and based on the situation this could be either impossible, or could involve some loss.

Suppose first that the $a_n$ are bounded. Then $$ B(x+h) - B(x) = \int_{x}^{x+h} A(t) \frac{dt}{t} = (A(x)+O(h)) \log\frac{x+h}{x}, $$ and choosing $h=x^{3/4}$ we obtain $A(x)=x+O(x^{\frac 34})$. This was noted in Alpoge's comment above, and also holds if the $a_n$ are given to be non-negative rather than bounded. However there is a loss in this argument and the $x^{3/4}$ error term cannot be improved, even for bounded non-negative $a_n$. Here is an example: Take $a_n=2$ if $n \in [m^4,(m^4+(m+1)^4)/2)$ for some integer $m$, and $a_n=0$ if $n\in ((m^4+(m+1)^4)/2,(m+1)^4)$. Then you can check that $B(x) = x +O(x^{\frac 12})$ holds, but no estimate sharper than $A(x) = x +O(x^{\frac 34})$ can hold.

Finally without some restrictions on $a_n$, one can say nothing at all. For example, take $a_n=1+ (-1)^n n$. Then you can check that $A(n)$ alternates between about $n/2$ and about $3n/2$, whereas $B(n)$ is very close to $n$.

$\endgroup$
1
  • $\begingroup$ thanks! (apologies for the few years' delay) $\endgroup$
    – tomos
    Jan 12, 2022 at 10:28

Your Answer

By clicking “Post Your Answer”, you agree to our terms of service and acknowledge you have read our privacy policy.

Not the answer you're looking for? Browse other questions tagged or ask your own question.